HSC 2014 MX2 Marathon ADVANCED (archive) (1 Viewer)

Status
Not open for further replies.

mreditor16

Well-Known Member
Joined
Apr 4, 2014
Messages
3,178
Gender
Male
HSC
2014
Re: HSC 2014 4U Marathon - Advanced Level

To be honest, I actually forgot to register lol. For some reason I had it in my head that it was next week :/.

No biggie though, I will just do the exam at home later whenever it gets uploaded.
me too :)
 

Chlee1998

Member
Joined
Oct 1, 2014
Messages
90
Gender
Male
HSC
2015
Re: HSC 2014 4U Marathon - Advanced Level

Given that a + b + c= 3

Prove that a^2 + b^2 + c^2 + ab+ bc+ca>=6
 

Sy123

This too shall pass
Joined
Nov 6, 2011
Messages
3,730
Gender
Male
HSC
2013
Re: HSC 2014 4U Marathon - Advanced Level

 

glittergal96

Active Member
Joined
Jul 25, 2014
Messages
418
Gender
Female
HSC
2014
Re: HSC 2014 4U Marathon - Advanced Level

Consider for . This function is concave up, and so every secant of its graph lies above the graph itself.

This means that for and , we have . (The sum 1 condition means that we are doing an internal division of a line segment.)

Now for a given and , we have





In other words we have



Applying this to the LHS of the inequality we are trying to prove and we get

 

Sy123

This too shall pass
Joined
Nov 6, 2011
Messages
3,730
Gender
Male
HSC
2013
Re: HSC 2014 4U Marathon - Advanced Level

Consider for . This function is concave up, and so every secant of its graph lies above the graph itself.

This means that for and , we have . (The sum 1 condition means that we are doing an internal division of a line segment.)

Now for a given and , we have





In other words we have



Applying this to the LHS of the inequality we are trying to prove and we get

Nice
My alternate solution:

 

Chlee1998

Member
Joined
Oct 1, 2014
Messages
90
Gender
Male
HSC
2015
Re: HSC 2014 4U Marathon - Advanced Level

(Assuming a,b,c are non-negative)









Just an alternative,
from (a+b+c)(1/a + 1/b+ 1/c) >=9, 1/a + 1/b + 1/c >=3

From AM GM, a+b+c/3 = 1> (abc)^1/3. Thus abc < 1

transform LHS into (a+b+c)^2 - ab -bc -ca.

which equals 9 - abc(1/a + 1/b + 1/c) >= 9-3 =6
 
Last edited:

RealiseNothing

what is that?It is Cowpea
Joined
Jul 10, 2011
Messages
4,591
Location
Sydney
Gender
Male
HSC
2013
Re: HSC 2014 4U Marathon - Advanced Level

from (a+b+c)(1/a + 1/b+ 1/c) >=9, 1/a + 1/b + 1/c >=3

From AM GM, a+b+c/3 = 1> (abc)^1/3. Thus abc < 1

transform LHS into (a+b+c)^2 - ab -bc -ca.

which equals 9 - abc(1/a + 1/b + 1/c) >= 9-3 =6
You barely proved half the things you claimed.
 

Chlee1998

Member
Joined
Oct 1, 2014
Messages
90
Gender
Male
HSC
2015
Re: HSC 2014 4U Marathon - Advanced Level

You barely proved half the things you claimed.

A fair criticism. However if you look at Sys previous post he didnt include a proof for am gm. So I assumed on this forum I was could assume the really common theorems. Of course in an exam i wouldve most definity included proofs. But i didnt think including them here would be necessary.
 
Last edited:

RealiseNothing

what is that?It is Cowpea
Joined
Jul 10, 2011
Messages
4,591
Location
Sydney
Gender
Male
HSC
2013
Re: HSC 2014 4U Marathon - Advanced Level

A fair criticism. However if you look at Sys previous post he didnt include a proof for am gm. So I assumed on this forum I was could assume the really common theorems. Of course in an exam i wouldve most definity included proofs. But i didnt think including them here would be necessary.
I wasn't referring to AM-GM.

for example

Also
 

Chlee1998

Member
Joined
Oct 1, 2014
Messages
90
Gender
Male
HSC
2015
Re: HSC 2014 4U Marathon - Advanced Level

I wasn't referring to AM-GM.

for example

Also


whoops i just realised the previous method didn't quite work. Thanks Realise.

But I just thought of another method.


Change the LHS into (3-c)^2 + (3-a) ^2 + (3-b)^2
The goal is to prove that this is greater than 12
After expanding and cancelling off terms, you are left with proving that a^2 + b^2 +c^2 > a+ b+ c
Which comes really simply from a^2 + 1 > 2a (AM GM)
and b^2 + 1 > 2b
c^2 + 1 > 2c

Suming all three of these gives a^2 + b^2 + c^2 + 3 > 2a + 2b + 2c
and considering that a+b+c =3,
a^2 + b^2 +c^2 > a+ b+c.

Hence proven.
I am open to your criticism Realise
 

glittergal96

Active Member
Joined
Jul 25, 2014
Messages
418
Gender
Female
HSC
2014
Re: HSC 2014 4U Marathon - Advanced Level

not sure which forum to post this. Hope this is ok
The number of orbits at such a time t must differ by integral amounts, so we have

t/p = t/p^2 + k

and

t/p^2 = t/p^3 + l

for some positive integers k,l. (Positive because their different periods means they definitely can't have had the same number of orbits at any positive time.)

Solving these for t, we get

t= kp^2/(p-1) = lp^3/(p-1).

As l must be at least 1, t must be at least p^3/(p-1). This is in fact possible, because we can take k=p to ensure that the middle quantity is also equal.

So the time sought is p^3/(p-1).
 

glittergal96

Active Member
Joined
Jul 25, 2014
Messages
418
Gender
Female
HSC
2014
Re: HSC 2014 4U Marathon - Advanced Level

part ii)?
Oh didn't see that! Does it just mean that the 3 are in a line, with some on opposite sides of the star? Will do this in a bit, just about to grab dinner :).
 

glittergal96

Active Member
Joined
Jul 25, 2014
Messages
418
Gender
Female
HSC
2014
Re: HSC 2014 4U Marathon - Advanced Level

yep that's right
Ah okay, then the same thing works really, you just have to replace the "differ by integers" with "differ by half-integers".

We get

which has minimal solution

 

abecina

Member
Joined
Aug 24, 2008
Messages
44
Gender
Male
HSC
N/A
Re: HSC 2014 4U Marathon - Advanced Level

Here's a follow up question.

A clock has an hour-hand of 3 units and a minute-hand of 4 units. If the hands are currently positioned at three-o'-clock, determine the first time when the tips of the hands are moving away from each other the fastest
 
Last edited:

glittergal96

Active Member
Joined
Jul 25, 2014
Messages
418
Gender
Female
HSC
2014
Re: HSC 2014 4U Marathon - Advanced Level

Here's a follow up question.

A clock has an hour-hand of 3 units and a minute-hand of 4 units. If the hands are currently positioned at three-o'-clock, determine the first time when the tips of the hands are moving away from each other the fastest
This is a bit tedious, so I won't include every detail, but I think this is right.

We can write the two hands endpoints in polar coordinates as



As the hands have constant angular speeds, and hand 2 is moving faster, the first time the tips are moving away the fastest will occur when



We write for brevity. (This is the angle between hands, offset by a constant factor of .)

If x is the separating distance, we have (using the cosine rule),



Differentiating twice using the chain rule we get



and



Now, at the time of interest, x'' vanishes, so plugging the expression for the first derivative of x into the second equation and simplifying we get



Use the Pythagorean identity to put everything in terms of sin(x), and factorise the resulting quadratic to get.



Since must lie in one of the second two quadrants (*), this gives



(Here t is the number of hours elapsed since the starting time of 3:00.)
 
Last edited:
Status
Not open for further replies.

Users Who Are Viewing This Thread (Users: 0, Guests: 1)

Top